Neliönjäännökset

Tässä postauksessa käsitellään neliönjäännöksiä. Tarkemmin sanoen käsitellään, milloin luku on neliönjäännös äärettömän monella alkuluvulla, sekä muita samanhenkisiä kysymyksiä.

Ensin kerrataan neliönjäännöksiin liittyvä teoria. Väitteitä ei todisteta tässä, ja kiinnostuneen lukijan kehotetaan siirtymään Esa Vesalaisen lukuteoriamonisteen pariin (luku 5).

Määritelnä (neliönjäännös)

Olkoon $p$ alkuluku. Sanotaan, että $a$ on neliönjäännös modulo $p$, jos $p \not\mid a$, ja yhtälöllä $x^2 \equiv a \pmod{p}$ on ratkaisu. Jos $p \not\mid a$, ja tällä yhtälöllä ei ole ratkaisua, kutsutaan $a$:ta neliönepäjäännökseksi. Luvut $a$, joilla $p \mid a$, eivät ole neliönjäännöksiä eivätkä -epäjäännöksiä.

Parittomilla $p$ neliönjäännöksiä ovat täsmälleen puolet luvuista $1, 2, \ldots , p-1$.

Määritelmä (Legendren symboli)

Olkoon Legendren symboli määritelty asettamalla $L(a, p) = 1$, jos $a$ on neliönjäännös modulo $p$, $L(a, p) = -1$ jos $a$ on neliönepäjäännös, ja muuten $L(a, p) = 0$.

Tässä postauksessa käytetään ulkonäkösyistä yllä esitettyä tavallisuudesta poikkeavaa merkintään Legendren symbolille.

Tämän postauksen kannalta tärkeimmät tulokset neliönjäännökset ovat seuraavat:

Multiplikatiivisuus

$L(ab, p) = L(a, p)L(b, p)$ kaikilla $a, b \in \mathbb{Z}$.

Neliönjäännösten resiprookkilaki

Olkoot $p$ ja $q$ parittomia alkulukuja. Oletetaan, että joko $p \equiv 1 \pmod{4}$ tai $q \equiv 1 \pmod{4}$. Tällöin $L(q, p) = L(p, q)$. Jos taas $p \equiv q \equiv 3 \pmod{4}$, pätee $L(q, p) = -L(p, q)$.

Resiprookkilaki ei kerro, milloin luvut $-1$ ja $2$ ovat neliönjäännöksiä. Näitä varten on seuraavat tulokset:

Tapaus $a = -1$

$L(-1, p) = 1$ jos ja vain jos $p \equiv 1 \pmod{4}$ (tai $p = 2$).

Tapaus $a = 2$

$L(2, p) = 1$ jos ja vain jos $p \equiv \pm 1 \pmod{8}$.

Näillä esitiedoilla pääsemme postauksen keskeisiin teemoihin.

Ongelma 1

Olkoon $a \neq 0$ kokonaisluku. Onko olemassa äärettömän monta alkulukua $p$, joilla $L(a, p) = 1$?

Ongelma 2

Olkoon $a \neq 0$ kokonaisluku. Onko olemassa äärettömän monta alkulukua $p$, joilla $L(a, p) = -1$?

On uskottavaa, että ongelmassa 1 vastaus on aina myönteinen. Ongelmassa 2 näin ei ole jos $a$ on neliöluku, mutta muissa tapauksissa vastaus lienee myönteinen. Intuitiiviset vastaukset ovat oikeat. Seuraavaksi esitetään todistukset tuloksille. Lukijan on hyvä koittaa ratkaista tehtäviä ennen ratkaisujen lukemista.

Ratkaisu 1

Ongelmassa tutkitaan sitä, milloin jokin luku on neliönjäännös mod $p$. Ideana on kääntää ongelma muotoon, jossa tutkitaan, milloin $p$ on neliönjäännös modulo jokin luku tai jotkin luvut. Tämä muoto on helpompi käsitellä, ja antaa ratkaisun ongelmaan.

Tutkitaan vain niitä alkulukuja $p$, joilla $p \equiv 1 \pmod{8}$. Näitä on Dirichlet’n lauseen nojalla äärettömän monta. Syynä tähän valintaan on se, että tällöin $L(-1, p) = L(2, p) = 1$, joten todistus yksinkertaistuu hieman.

Lemma 1

Olkoon $k \in \mathbb{Z}$ sellainen, jolla $k \neq 0$. Olkoon $\mid k \mid \ = 2^{a}p_1^{a_1}p_2^{a_2} \ldots p_n^{a_n}q_1^{b_1} \ldots q_m^{b_m}$, missä parittomat alkuluvut $p_i, q_j$ ovat erisuuria, $a_i \equiv 0 \pmod{2}$ ja $b_j \equiv 1 \pmod{2}$. Tällöin kaikilla $p \equiv 1 \pmod{8}$, joilla $p \not\mid k$, pätee

$L(k, p) = L(q_1, p)L(q_2, p) \ldots L(q_m, p)$

Huomautus

Jos $\mid k \mid$ on luvun $2$ potenssi, on kyseessä tyhjä tulo, eli $L(k, p) = 1$.

Todistus

Ideana on käyttää Legendren symbolin multiplikatiivisuutta, jonka avulla ongelma voidaan hajoittaa pieniin palasiin. Valinnasta $p \equiv 1 \pmod{8}$ ja multiplikatiivisuudesta johtuen

Tätä voidaan sieventää seuraavin keinoin:

  1. Koska $p \equiv 1 \pmod{8}$, pätee $L(2^a, p) = L(2, p)L(2, p) \ldots L(2, p) = 1^a = 1$.

  2. Luvut $p_i^{a_i}$ ovat neliölukuja, koska $a_i \equiv 0 \pmod{2}$. Siis ne ovat neliönjäännöksiä modulo $p$, joten $L(p_i^{a_i}, p) = 1$ kaikilla $i$.

  3. $b_i \equiv 1 \pmod{2}$, joten voidaan kirjoittaa $b_i = 2c_i + 1$, $c_i \in \mathbb{Z}$. Tällöin $L(q_i^{b_i}, p) = L(q_i^{2c_i}q_i, p) = L(q_i^{2c_i}, p)L(q_i, p)$. Mutta $q_i^{2c_i}$ on neliöluku, joten tämä sieventyy muotoon $L(q_i, p)$.

Edellisten huomioiden nojalla saadaan $L(k, p) = L(q_1, p) \ldots L(q_m, p)$. Tämä todistaa lemman väitteen.

Ratkaisun 1 viimeistely

Lemman avulla voidaan siirtyä ratkaisun alussa kuvailtuun vaiheeseen, jossa käännetään symbolin $L(a, p)$ tutkiminen siihen, milloin $p$ on neliönjäännös. Kirjoitetaan $L(a, p) = L(q_1, p) \ldots L(q_m, p)$, missä $q_i$ ovat parittomia alkulukuja. Koska valinnan mukaan $p \equiv 1 \pmod{8}$, on $p \equiv 1 \pmod{4}$, joten voimme käyttää neliönjäännösten resiprookkilakia ilman etumerkkien vaihtumista:

Enää tulee siis varmistaa, että yhtälön oikeanpuoleisin lauseke on $1$. Mutta tämä on helppoa: voidaan esimerkiksi valita $p \equiv 1 \pmod{8q_1q_2 \ldots q_m}$. Tällöin $p \equiv 1 \pmod{8}$ kuten kuuluukin. Lisäksi tärkeimpänä huomiona $p \equiv 1 \pmod{q_i}$ kaikilla $i$, joten $p$ on neliönjäännös modulo $q_i$ kaikilla $i$. Tämä osoittaa, että kaikki tätä muotoa olevat $p$ (poislukien ne, joilla $p \mid a$) ovat sellaisia, joilla $L(a, p) = 1$. Dirichlet’n lauseen nojalla näitä $p$ on äärettömän monta, mikä todistaa väitteen.

Huomaa, että ongelma 1 voidaan ratkaista suoraan Schurin lauseella. Yllä esitetty ratkaisu on kuitenkin tietyssä mielessä parempi, koska se antaa informaatiota siitä, “millaisia” ne $p$ ovat, joilla $L(a, p) = 1$. Tähän keskitytään tarkemmin myöhemmin tässä postauksessa.

Esitetään sitten vastaavassa hengessä ratkaisu ongelmaan 2.

Ratkaisu 2

Ratkaisu tähän ongelmaan ei poikkea suuresti edellisen tehtävän ratkaisusta, mutta vaikeutena on se, että edellisessä ongelmassa ainoa erikoinen tapaus oli $a = 0$, ja tässä eri tavalla käyttäytyviä lukuja on äärettömän monta.

Ongelman 1 ratkaisu antaa miltei suoraan ratkaisun siinä tapauksessa, että $\pm a$:lla on sellainen pariton alkulukutekijä $q$, jonka eksponentti $\pm a$:n alkutekijähajotelmassa on pariton. Tutkitaan tämä tapaus ensimmäisenä.

Olkoon siis ratkaisun 1 mukaisesti $p \equiv 1 \pmod{8}$, ja $L(a, p) = L(q_1, p)L(q_2, p) \ldots L(q_m, p)$. Oletuksen nojalla $m \ge 1$, eli kyseessä ei ole tyhjä tulo. Jälleen voidaan käyttää neliönjäännösten resiprookilakia:

Koska $q_1$ on pariton alkuluku, on olemassa sellainen $t \in \mathbb{Z}$, jolla $L(t, q_1) = -1$. Valitaan nyt $p$ niin, että seuraavat ehdot toteutuvat:

  1. $p \equiv 1 \pmod{8}$
  2. $p \equiv t \pmod{q_1}$
  3. $p \equiv 1 \pmod{q_i}$ kaikilla $2 \le i \le m$. Jos $m = 1$, ei tätä ehtoa tarvitse huomioida.

Kiinalaisen jäännöslauseen ja Dirchlet’n lauseen nojalla tällaisia $p$ on äärettömän monta. Näillä $p$ pätee $L(p, q_i) = 1$ jos ja vain jos $i \neq 1$. Siis täsmälleen yksi tulon $L(p, q_1)L(p, q_2) \ldots L(p, q_m)$ luvuista on $-1$, joten koko tulo on $-1$. Tämä osoittaa väitteen.

Tutkitaan sitten se tapaus, jossa $\mid a \mid$:lla ei ole parittomia alkulukutekijöitä. Tämä tarkoittaa, että $\mid a \mid = 2^b$ jollain $b \ge 0$. Siis $a = \pm 2^b$. Tapaus $b \ge 2$ voidaan redusoida lemman 1 todistuksen kaltaisilla menetelmillä tapaukseen $0 \le b < 2$. Näiden äärellisen monen tapauksen käsittely jätetään lukijalle.

Yleistäminen

Todistetut tulokset ovat varsin kauniita, mutta voiko niitä vielä laajentaa? Luonnollisia kysymyksiä ovat esimerkiksi seuraavat:

  1. Päteekö ongelman 1 variantti, jossa vaaditaan $L(a_i, p) = 1$ kaikilla $i$ ja äärettömän monella $p$, missä $a_1, a_2, \ldots a_n$ ovat annettuja nollasta eroavia kokonaislukuja?

  2. Entä ongelman 2 vastaava variantti, missä $a_i$ eivät ole neliölukuja?

  3. Ratkaisuissa 1 ja 2 luodut $p$ ovat melko harvassa, esimerkiksi ratkaisun 1 $p$ toteuttavat ehdon $p \equiv 1 \pmod{8q_1q_2 \ldots q_m}$. Kuinka “tiheästi” ongelman 1 tai 2 mukaisia $p$ todellisuudessa on?

Ensimmäisen kysymyksen vastaus on melko ilmeinen: käytännössä sama argumentti käsittelee monen muuttujan tapauksen kuin mitä käytettiin ratkaisussa 1.

Kysymyksen 2 vastaus ei ole niin ilmeinen: olkoot $a_1 = 2 \cdot 3$, $a_2 = 3 \cdot 5$ ja $a_3 = 2 \cdot 5$. Jos olisi olemassa $p$, jolla $L(a_i, p) = -1$ kaikilla $i$, niin pätisi $-1 = L(a_1, p)L(a_2, p)L(a_3, p) = L(a_1a_2a_3, p) = L((2\cdot3\cdot5)^2, p) = 1$. Tämä on selvästi mahdotonta. Yleisesti voidaan todistaa vastaavasti, että minkään parittoman määrän lukuja $a_i$ tulo ei saa olla neliöluku. Onko tämä riittävä ehto, eli jos tämä ehto täyttyy, niin onko olemassa äärettömän monta kelpaavaa $p$? Osoittautuu, että on. Heuristinen selitys tälle esitetään kysymyksen 3 käsittelyn jälkeen. Kirjoittajan formaali todistus on liian pitkä tähän postaukseen, mutta se saatetaan nähdä myöhemmässä postauksessa.

Kysymys 3

Kysymys 3 ei ole mielenkiintoinen neliölukujen $a$ tapauksessa: tällöin $a$ on neliönjäännös kaikilla $p$. Muussa tapauksessa intuitiivinen veikkaus voisi olla, että “puolet” alkuluvuista on sellaisia, että $a$ on neliönjäännös, ja loput puolet ovat sellaisia, joilla $a$ on neliönepäjäännös, koska jokaista $p$ kohden puolet luvuista ovat neliönjäännöksiä ja puolet eivät.

Ennen tämän väitteen todistamista määritellään tiheys. Olkoon $S$ jokin alkulukujen $\mathbb{P}$ osajoukko. Merkitään joukon $S \subset \mathbb{P}$ lukua $x$ pienempien alkioiden määrä $S(x)$, ja merkitään vastaavasti $\mathbb{P}(x)$. Joukon $S$ tiheys määritellään raja-arvona

Kaikilla joukoilla $S$ tämä raja-arvo ei ole olemassa, mutta tässä postauksessa tutkittavat tiheydet ovat olemassa. Dirichlet’n lauseesta on usein kilpailumatematiikan yhteydessä mainittua versiota voimakkaampi väite, joka kertoo, että niiden $p$ tiheys, joilla $p \equiv a \pmod{m}$, on $\frac{1}{\phi(m)}$ kaikilla $a, m$, jotka ovat yhteistekijättömiä.

Käyttäen Dirichlet’n lausetta ei ole kovin vaikeaa osoittaa ratkaisun 1 kaltaisesti, että niiden $p$ tiheys, joilla $L(a, p) = 1$, on $\frac{1}{2}$. Tämä voidaan toteuttaa vaikkapa seuraavasti:

Ttkitaan ensin niitä $p$, joilla $p \equiv 1 \pmod{8}$. Näitä $p$ on neljäsosa alkuluvuista, ja näillä $p$ pätee $L(a, p) = L(p, q_1) \ldots L(p, q_m)$.

Olkoon $T = (t_1, t_2, \ldots , t_m)$ jokin $m$ luvun jono, joka koostuu luvusta $-1$ ja $1$. Kiinalaisen jäännöslauseen ja Dirichlet’n lauseen tiheysversion avulla saadaan, että niiden $p$ tiheys, joilla $L(p, q_i) = t_i$ kaikilla $i$ on $2^{-m}$. Niitä $T$, jotka sisältävät parillisen määrän lukua $-1$ on $2^{m-1}$ kappaletta. Tästä saadaan, että yhteensä niitä $p$, joilla $L(a, p) = 1$ on tapauksessa $p \equiv 1 \pmod{8}$ puolet alkuluvuista.

Tutkiessa tapauksia $p \equiv 3, 5, 7 \pmod{8}$ päädytään yhtälöön $L(a, p) = \pm L(p, q_1) \ldots L(p, q_m)$, missä etumerkki riippuu vain siitä, mitä $p$ on modulo $8$. Tällöin saadaan edellisen argumentin kaltaisesti jokaisen tapauksen alkutekijöiden tiheydelle arvo $\frac{1}{2}$. Siispä kokonaisuudessaan niitä $p$, joilla $L(a, p) = 1$, on puolet alkuluvuista.

Huomautus

Missä kohtaa käytettiin tietoa siitä, ettei $a$ ole neliöluku?

Kysymys 2

Perustellaan ensiksi se, että riittää tutkia tapausta, joissa kertomalla lukuja $a_i$ sopivasti keskenään ei saa neliölukua. Nimittäin, oletetaan että lukujen $a_1, a_2, \ldots , a_t$ tulo on neliöluku $T^2$. Oletuksen nojalla $t$ on parillinen. Siis $a_1a_2 \ldots a_t = T^2$, joten $a_1^2a_2 a_3 \ldots a_t = T^2a_1$. Ottamalla Legendren symbolin puolittain saadaan, että

Siis jos $a_2, a_3, \ldots , a_t$ ovat kaikki neliönepäjäännöksiä, seuraa tästä $L(a_1, p) = -1$, koska $t$:n oletettiin olevan parillinen.

Tämä idea soveltuu neliönepäjäännösten lisäksi tietysti myös neliönjäännösten tutkimiseen. Tehdään tämä oletus, ja esitetään heuristiikka, johon kysymyksen 2 ilmiö perustuu.

Heuristiikka

Olkoot $a_1, a_2, \ldots , a_n$ kokonaislukuja. Oletetaan, että ei ole olemassa epätyhjää osajoukkoa näistä luvuista, jonka alkioiden tulo olisi neliöluku. Heuristisesti niiden $p$ tiheys, joilla $L(a_i, p) = 1$ kaikilla $i$, on $2^{-n}$.

Perustelu

Väite todistettiin kysymyksen 3 tapauksessa arvolla $n = 1$. Tutkitaan konkreettisuuden vuoksi tapausta $n = 2$, $a_1 = 2$, $a_2 = 3$. Tiedetään, että puolet alkuluvuista ovat sellaisia, joilla $2$ on neliönjäännös, ja vastaavasti luvulle $3$. Ei ole ilmeistä syytä, miksi nämä tapahtumat riippuisivat toisistaan: tieto siitä, että $2$ on neliönjäännös ei vaikuta kertovan mitään siitä, onko $3$ neliönjäännös vai ei. Voidaan siis ajatella, että tapahtumat “$2$ on neliönjäännös” ja “$3$ on neliönjäännös” ovat toisistaan riippumattomia, ja tapahtuvat molemmat $50\%$ ajasta. Siis neljäsosa alkuluvuista on sellaisia, joilla $2$ ja $3$ molemmat ovat neliönjäännöksiä modulo $p$.

Yleinen tapaus voidaan perustella vastaavasti.

Heuristiikkaa voi soveltaa aivan vastaavasti myös neliönepäjäännöksille (ja myös sekoituksille jäännöksiä ja epäjäännöksiä). Tämä perustelee väitteen.

Huomautus

Lienee mahdollista osoittaa heuristiikan paikkansapätevyys formaalisti kysymyksen 3 ratkaisun kaltaisin menetelmin. Erikoistapauksia voi ratkoa kohtuullisen yksinkertaisesti (esimerkiksi se tapaus, että $a_i$ ovat erisuuria alkulukuja), mutta yleinen tapaus vaatii uusia ideoita.

Harjoitustehtäviä

Yleinen toisen asteen polynomi

Olkoon $P(x) = ax^2 + bx + c$ kokonaislukukertoimien toisen asteen polynomi. Osoita, että äärellisen montaa $p$ lukuun ottamatta yhtälöllä $P(x) \equiv 0 \pmod{p}$ on ratkaisu jos ja vain jos $b^2 - 4ac$ on neliönjäännös modulo $p$ (tai $b^2 - 4ac \equiv 0 \pmod{p}$).

Erikoistapaus

Olkoot $a_1, a_2, \ldots , a_n$ pareittain yhteistekijättömiä positiivisia kokonaislukuja, jotka eivät ole neliölukuja. Olkoon $S$ jokin osajoukko joukosta $\lbrace 1, 2, \ldots , n \rbrace$, ja olkoon $T$ niiden $p$ joukko, joilla $L(a_i, p) = 1$ jos ja vain jos $i \in S$. Osoita, että $\delta(T) = 2^{-n}$.

Korkeammat potenssit

Neliönjäännöksiä on puolet luvuista tiettyä $p$ kohden, joten heuristisesti $a$ on neliönjäännös modulo puolet alkuluvuista, ellei alkuluvut “suosisi” $a$:ta olemaan neliönjäännös. Tämä tapahtuu postauksen tulosten nojalla täsmälleen silloin, kun $a$ on neliöluku, eli vain ilmeisessä tapauksessa.

Esitä heuristinen perustelu sille, että $a$ on kuutionjäännös modulo $p$ (eli yhtälöllä $x^3 \equiv a \pmod{p}$ on ratkaisu) $\frac{2}{3}$ alkuluvuista $p$ kun $a$ ei ole kuutioluku. Esitä vastaava perustelu eksponentin $3$ sijasta alkulukueksponentille $q$, jolloin tiheyden tulisi olla $\frac{q-1}{q}$.

Huomautetaan, että heuristiikat todella antavat oikean tuloksen, mutta väitteiden todistaminen vaatii järeämpää kalustoa. Vinkkinä tehtävään toimii tämän monisteen aihe.